Jiasheng Jin
Jiasheng Jin
  • Видео 218
  • Просмотров 177 498

Видео

A nice estimation of (1+1/n)^n
Просмотров 12614 дней назад
You can leave a comment down below, like and subscribe to my channel if you like my content. You can also support me on Ko-fi at ko-fi.com/jiashengjin
Can you find the minimum value?
Просмотров 15721 день назад
You can leave a comment down below, like and subscribe to my channel if you like my content. You can also support me on Ko-fi at ko-fi.com/jiashengjin
Have you ever seen this proof before?
Просмотров 106Месяц назад
You can leave a comment down below, like and subscribe to my channel if you like my content. You can also support me on Ko-fi at ko-fi.com/jiashengjin
Number theory problem | 133^5+110^5+84^5+27^5=n^5
Просмотров 5114 месяца назад
You can leave a comment down below, like and subscribe to my channel if you like my content. You can also support me on Ko-fi at ko-fi.com/jiashengjin
Rough estimation of 1+2^2+3^3+…+n^n
Просмотров 1914 месяца назад
You can leave a comment down below, like and subscribe to my channel if you like my content. You can also support me on Ko-fi at ko-fi.com/jiashengjin
When is this an integer? n^1/n-7 | Swedish Math Olympiad 2002
Просмотров 2,4 тыс.4 месяца назад
You can leave a comment down below, like and subscribe to my channel if you like my content. You can also support me on Ko-fi at ko-fi.com/jiashengjin
Some hard analysis | limit of (an+1+an+2)/an
Просмотров 4274 месяца назад
You can leave a comment down below, like and subscribe to my channel if you like my content. You can also support me on Ko-fi at ko-fi.com/jiashengjin
Iran Math Olympiad number theory problem | x^13 equals 21982145917308330487013369
Просмотров 28 тыс.5 месяцев назад
You can leave a comment down below, like and subscribe to my channel if you like my content. You can also support me on Ko-fi at ko-fi.com/jiashengjin
Does this limit exist? | sqrt(1+sqrt(2+…+sqrt(n)))
Просмотров 3805 месяцев назад
You can leave a comment down below, like and subscribe to my channel if you like my content. You can also support me on Ko-fi at ko-fi.com/jiashengjin
A floor function equation | floor of sqrtn+1/2 floor of sqrt(n-3/4)+1/2
Просмотров 1565 месяцев назад
You can leave a comment down below, like and subscribe to my channel if you like my content. You can also support me on Ko-fi at ko-fi.com/jiashengjin
IMO 2020 Question 2
Просмотров 3065 месяцев назад
You can leave a comment down below, like and subscribe to my channel if you like my content. You can also support me on Ko-fi at ko-fi.com/jiashengjin
Harmonic series | generalized Abel second theorem
Просмотров 1855 месяцев назад
You can leave a comment down below, like and subscribe to my channel if you like my content. You can also support me on Ko-fi at ko-fi.com/jiashengjin
Tricky inequality | double sum (aiaj/ci+cj) (bibj/ci+cj) bigger than (aibj/ci+cj)^2
Просмотров 1745 месяцев назад
You can leave a comment down below, like and subscribe to my channel if you like my content. You can also support me on Ko-fi at ko-fi.com/jiashengjin
Some integral inequality
Просмотров 2245 месяцев назад
You can leave a comment down below, like and subscribe to my channel if you like my content. You can also support me on Ko-fi at ko-fi.com/jiashengjin
A mysterious floor equation identity
Просмотров 1695 месяцев назад
A mysterious floor equation identity
Last 2 digits of 9^9^9^9
Просмотров 3405 месяцев назад
Last 2 digits of 9^9^9^9
How on earth can we show the convergence radius is 1?
Просмотров 1345 месяцев назад
How on earth can we show the convergence radius is 1?
MIT Integration Bee 2024 problem 4
Просмотров 2055 месяцев назад
MIT Integration Bee 2024 problem 4
3 ways to solve this MIT Integration Bee problem | cosx+xsinx/x(x+cosx)dx
Просмотров 4945 месяцев назад
3 ways to solve this MIT Integration Bee problem | cosx xsinx/x(x cosx)dx
A fabulous integral from MIT Integration Bee | infinite nested roots
Просмотров 1,5 тыс.5 месяцев назад
A fabulous integral from MIT Integration Bee | infinite nested roots
A problem from Analysis | limsupn((1+an+1/an)-1) no less than 1
Просмотров 2315 месяцев назад
A problem from Analysis | limsupn((1 an 1/an)-1) no less than 1
How this hard infinite sum equals pi/2ln2
Просмотров 4445 месяцев назад
How this hard infinite sum equals pi/2ln2
A tricky integral from 0 to pi/2 of lnsinxdx
Просмотров 1616 месяцев назад
A tricky integral from 0 to pi/2 of lnsinxdx
Is it 7 or any number is correct?
Просмотров 2596 месяцев назад
Is it 7 or any number is correct?
Integral of dx/1+acosx
Просмотров 3726 месяцев назад
Integral of dx/1 acosx
This cannot be true! | What went wrong?
Просмотров 1196 месяцев назад
This cannot be true! | What went wrong?
A tricky limit problem | lim(a2n+2an)=0
Просмотров 2686 месяцев назад
A tricky limit problem | lim(a2n 2an)=0
No integral by parts allowed | integral from 0 to 1 of lnxdx
Просмотров 7156 месяцев назад
No integral by parts allowed | integral from 0 to 1 of lnxdx
Some inequalities of liminf and limsup
Просмотров 1636 месяцев назад
Some inequalities of liminf and limsup

Комментарии

  • @obrisaow7392
    @obrisaow7392 День назад

    nice blackboard

  • @jjeastside
    @jjeastside День назад

    thx

  • @Nikhil-gh7qr
    @Nikhil-gh7qr День назад

    Can you solve this question f is real valued function. f^3(x)+f(x)=x. Find integration of f(x) from 0 to 2.

    • @jiashengjin
      @jiashengjin День назад

      let f(x)=t, following the given equation we have t^3+t=x, differentiate both sides, we have dx=(3t^2+1)dt, again from t^3+t=x, we plug 0 and 2 into x, we get t=0 and 1 as real solutions, so our new bounds are from 0 to 1, and integral becomes tdx=3t^3+tdt (to be rigorous, f(x) can be proven to be integrable from the cubic roots formula)

  • @puertavideo
    @puertavideo 2 дня назад

    Very good video

  • @madarauchiha4628
    @madarauchiha4628 2 дня назад

    appriciated

  • @FireFriendsGaming
    @FireFriendsGaming 12 дней назад

    Thankyou sir I am an Indian high school students

  • @yasharnamdar473
    @yasharnamdar473 18 дней назад

    Amazing. Is there a non creative solution that one can come up with on the spot?

  • @irfanteddiesvideos6664
    @irfanteddiesvideos6664 20 дней назад

    2nd Method- 🌞☀🌄 1st Method- 👹

  • @HildegardeDrda-h9o
    @HildegardeDrda-h9o 24 дня назад

    Mueller Via

  • @eveeeon341
    @eveeeon341 28 дней назад

    The statement requires that P be prime, so it's necessary to use that P is prime in the proof, but I'm struggling to see where it was used.

    • @mustafamotiwala2335
      @mustafamotiwala2335 26 дней назад

      We use it to show that k=p. We assume otherwise so that 1 < k < p and conclude from this that p = qk + r where r > 0. This is justified because p is prime and has no proper divisor greater than 1 (and k > 1). If p were not prime, this step would fail.

    • @eveeeon341
      @eveeeon341 26 дней назад

      ​@@mustafamotiwala2335 ahh of course, thank you for that!

  • @oxpie
    @oxpie 29 дней назад

    I took pre college maths as optional in my tuition and they taught this there

  • @Unacomputoentrecicleros
    @Unacomputoentrecicleros Месяц назад

    This channel is amazing

  • @SaiBekit
    @SaiBekit Месяц назад

    Amazing! Thank you.

  • @aliali-i2z5q
    @aliali-i2z5q Месяц назад

    We have a lemma says that If lim an+1/an =L<1 with an >0 for all n Thus lim an =0 Since lim an+1/an=lim A/n+1=0 Thus lim an =0

  • @aliali-i2z5q
    @aliali-i2z5q Месяц назад

    How could epsilon be less then 2|a| and 4|a| at the same time Since we said for all epsilon if someone choose epsilon 12 and our a is 2

    • @jiashengjin
      @jiashengjin 29 дней назад

      Since we implicitly assume epsilon is ‘small enough’

  • @aliali-i2z5q
    @aliali-i2z5q Месяц назад

    It's similar of using Cauchy sequence to prove that is convergent

  • @Jancel705
    @Jancel705 Месяц назад

    this is the best youtube proof on this

  • @mathdrmath9988
    @mathdrmath9988 2 месяца назад

    Excellent video Original and good proof We can also prove it using Jensen inequality and AM-GM inequality

  • @Aman_iitbh
    @Aman_iitbh 2 месяца назад

    Niceee

  • @heu-chernglin450
    @heu-chernglin450 2 месяца назад

    黑板寫得亂七八糟,講得模模糊糊,難怪觀看數那麼少。這題很簡單啊,答案心算就出來了。

  • @Manisha_Kharad
    @Manisha_Kharad 3 месяца назад

    Thankyou

  • @joseluishablutzelaceijas928
    @joseluishablutzelaceijas928 3 месяца назад

    Quite nice solution!

  • @joseluishablutzelaceijas928
    @joseluishablutzelaceijas928 3 месяца назад

    How can tan(x)/2 be a solution if tan is not continuous on pi/2 + pi*k for arbitrary integer values of k and one is asking for solutions continuous on the whole real line?

  • @joseluishablutzelaceijas928
    @joseluishablutzelaceijas928 3 месяца назад

    Thank you for the problem and its solution. I think one could alternatively observe that (1-x) = (1-x^(1/k))*(1+x^(1/k)+x^(2/k)+x^(3/k)+ ... +x^((k-1)/k)) for an arbitrary positive integer k and replace each of the factors in the denominator by such products, which leads to the initial expression being equal to 1/((1+x^(1/2))*(1+x^(1/3)+x^(2/3))* ... (1+x^(1/n)+x^(2/n)+x^(3/n)+ ... +x^((n-1)/n))). After letting x tend to 1 one then obtains 1/(2*3 ... *(n)) = 1/(n!).

  • @AnujYadav-wo6yf
    @AnujYadav-wo6yf 3 месяца назад

    Thanks

  • @Alex-ri6be
    @Alex-ri6be 4 месяца назад

    I like the use of that lemma.

  • @prism223
    @prism223 4 месяца назад

    I actually guessed the correct answer using a few memorized logarithms: The base ten logs for 1-10 are approximately 0, 0.3, 0.48, 0.6, 0.7, 0.78, 0.85, 0.9, 0.95, 1 Looking at the big number, it's basically 22x10^24, so its base 10 log is 24 + log(11) + 0.3. The 11 log isn't in the list I memorized, so I used linear interpolation between 12 and 10: log(12) ~ 1.08, so log(11) ~ 1.04. Then log(big number) ~ 25.34. Dividing by 13 to get the 13th root logarithm, I just do 3 digits mentally to get 1.94. This is extremely close to what I would compute for log(90) ~ 1.95, so my first guess would be that the 13th root of big number is 89.

  • @amirm1266
    @amirm1266 4 месяца назад

    Hello from iran

  • @RealQuInnMallory
    @RealQuInnMallory 4 месяца назад

    (x ➖ 3x+2)

  • @lechaiku
    @lechaiku 4 месяца назад

    Another approach without using modulo and log (this is "no-pen-no-paper" method): 1. the last digit of x must be 9, because only 9 goes in cycles (9,1) with the 13th power being 9. 2. 100^13 is 27-digit number, so x must be 2-digit number ----> 26(digits) : 13 = 2. 3. x can't be 99, because the given 26-digit number is too small (it's obvious). 4. 8 = (2^3)^13 = 2^39; let's calculate 2^40 5. 2^10 = 1024, let's use "1000" 6. 2^40 = approx. 8^13 = approx 1000 ^4 7. 80^13 = approx 1000 ^4 * 10^13 = 10^12 * 10^13 = 10^25 8. It is less than the given 26-digit number 9. therefore x = 89

  • @AnmolSahu
    @AnmolSahu 4 месяца назад

    1. Since 100^23 has 27 digits, x must be a two digit number. 2. Last digit of x^(4*3+1) must be same as last digit of x^1, therefore, last digit of x must be 9. 3. Digital sum of the given number is 8, therefore, the digital sum of x^13 must be 8. Solving which we get x=89.

  • @krule3753
    @krule3753 4 месяца назад

    i think you can stop and show contradiction at M(1/2)^4 < M(1/2)^k, since from the observation its implied that k > 4 but obviously in that case the inequality does not hold.

    • @asdasdasasdas5837
      @asdasdasasdas5837 4 месяца назад

      not really we have a_n<M for every n for example choose epsilon=1/2 then a_n<1/2(a_n+1 + a_n+2) but this doesnt imply M<1/2*(M+M) or M<M I hope you understand where im going with this, i really want to articulate more but english is my second language and i dont really know how to put it

  • @sie_khoentjoeng4886
    @sie_khoentjoeng4886 4 месяца назад

    Value of X must be odd, since the result is odd, and X is not 1 or 5 (where the result has unit 1 and 5) For K=1,2,3,4,5,..., then ?3^K, the unit result is 3,9,7,1,3,9,7,1,... ?7^K, the unit result is 7,9,3,,1,7,9,3,1,... ?9^K the unit result is 9,1,9,1,9,1,9,.... For K=13, then ?3^K has unit 3, ?7^K has unit 7 and ?9^K has unit 9 So, possible answer is 09,19,29,39,49,59,69,79,89,99 Number of digit 99^13 is about 36 (less than 36) so 99 is rejected, then X=88 (** if there is no typo like this 21,982,145,917,038,330,487,013,369 😊😊😊)

  • @tylosenpai6920
    @tylosenpai6920 4 месяца назад

    I just thought... Ends with 9 -> The original number must end with 3, 7, or 9 3 -> Final digits of 3 9 7 1 that repeats itself, will be 3 in 13 (Fail) 7 -> Final digits of 7 9 3 1 that repeats itself, will be 7 in 13 (Fail) 9 -> Final digits of 9 1 that repeats itself, will be 9 in 13 (Success) There are 26 digits, close to 10^26 with 27 digits (AKA 100^13), but not too close I'm now left with 89 and 99 as the most likely answers, and thought, "It shouldn't be too close" and picked 89...i was right...

  • @ВалентинПивоваров-о9с

    Last digit is 9, it's easy. 80^13 < 10^25, so it can be 99 or 89. If it 99 then the given number mast be divided by 11. Easy to se that it doesn't. So it is only 89

  • @orionspur
    @orionspur 4 месяца назад

    Rather easy to check that by sheer size x it has to be between 80 and 90 and has to end with 3 7 or 9. Of which only 9 can work. So x is 89.

  • @samueldeandrade8535
    @samueldeandrade8535 4 месяца назад

    For people who doesn't know, this equation was the first counterexample, to Euler's conjecture: If the sum of n many kth powers of positive integers is itself a kth power, then n≥k This conjecture was proved wrong in 1966, by L. J. Lander and T. L. Parkin. Their example was 27⁵+84⁵+110⁵+133⁵ = 144⁵

  • @ravTubeable
    @ravTubeable 4 месяца назад

    0:08 X is a real number 0:18 X is an integer number WTF!? How did you figured out (in just 10 seconds) that solution is an integer number?

  • @samueldeandrade8535
    @samueldeandrade8535 4 месяца назад

    Despite your content not being serious, it is very enjoyable to watch you.

  • @samueldeandrade8535
    @samueldeandrade8535 4 месяца назад

    Hahahahahahaha.

  • @meadbert
    @meadbert 4 месяца назад

    I counted digits and figured it had to be 2 digit number. Last digit had to be 9. Then I compared to 100 and saw we ended up at 22% of what 100 would have. This means we haved a little over two times so a half life of roughly 6 years. Law of 72 says we are decreasing by 12% per year to have a half life of six years so answer should be close to 88. Since last digit is 9 the answer must be 89.

    • @notmymain2256
      @notmymain2256 4 месяца назад

      Oh no finance has tainted modular arithmetic

  • @havenfractal
    @havenfractal 4 месяца назад

    10<x<100, since 10^13 would be too small and 100^13 is too big. x cannot end with an even, 1, or 5, because then x^13 would end in an even, 1, or 5, respectively. 7^2 and 3^2 end in 9, so 7^4 and 3^4 end in 1, so 7^12 and 3^12 end in 1. So 7^13 ends in 7 and 3^13 ends in 3. So the last digit cannot be 7 or 3. 9^2 ends in 1, so 9^12 ends in 1. So 9^13 ends in 9. The last digit can be 9. So we know that x is in the form 10y+9, with 0<y<10. Looking at the first 13 digits, y^13 <= 2198214591730, or about 2.198 * 10^12 because if it was bigger, the most significant 13 digits would have to be bigger. 7^2 < 50, so 7^12 < 50^6. So 7^13 < 50^6*7, or 1.09375 * 10^11. This is far too small. 8^3 = 2^9 > 500. So 8^12 > 500^4, and 8^13 > 5^4 * 8 * 10^8. So 8^13 > 5*10^11, which is in the right ballpark. 9^2 = 81 > 80, and 9^13 > 80^6 * 9 = 8^6 * 9 * 10^6 > 500^2 * 9 * 10^6, or 25*9 * 10^10, or 2.25*10^12, which is too big. So y=7 is proven too small, y=9 is proven too big, and y=8 looks plausible. So the answer is 89.

  • @ОлександрБереза-ю2м
    @ОлександрБереза-ю2м 4 месяца назад

    Прикольно

  • @cloudy7989
    @cloudy7989 4 месяца назад

    Strangely, x^13 = my phone number multipled by my wife's number plus our door number to the power of the numbers in my post code.

  • @lucabattazzi
    @lucabattazzi 4 месяца назад

    Thank you very much for this proof. There aren't many out there actually proving the existence of the limit, by using lim inf and lim sup

  • @shashigoodyiue
    @shashigoodyiue 4 месяца назад

    Gonna guess like 91?

  • @amarnathprasad1
    @amarnathprasad1 4 месяца назад

    Sir, doesn't this depend on the choice of e? We could choose e>1/2, and then a_n <= M(2e)^k -> inf, implying a_n is unbounded. Am I missing something here?

    • @krule3753
      @krule3753 4 месяца назад

      it should hold for ALL epsilon > 0, so no matter what epsilon we pick it should hold. since it doesn't work for epsilon = 1/4, then the "all" part doesn't hold, contradiction.

    • @amarnathprasad1
      @amarnathprasad1 4 месяца назад

      @@krule3753 Ohh, I see... Thanks for the explanation.

  • @ISTP-TCatboy
    @ISTP-TCatboy 4 месяца назад

    It's obviously 89. Because 9^9 has the last digit of 89 and same as 9^(10n+9). Which means that 9^9^9^9=9^9^(....89)=9^(....89)=.....89.

  • @ISTP-TCatboy
    @ISTP-TCatboy 4 месяца назад

    89

  • @atharvanaik5966
    @atharvanaik5966 4 месяца назад

    some classic hard anal. great video